Can someone help me with this please! I really need it done PLS

increase the amount of the ingredients by 5 and by 20

Can Someone Help Me With This Please! I Really Need It Done PLS Increase The Amount Of The Ingredients

Answers

Answer 1

Answer:

it's 50 g

Step-by-step explanation:

it's like a multiplying assberry kehe


Related Questions

two equal rectangular lots are enclosed by fencing the perimeter of a rectangular lot and then putting a fence across its middle. if each lot is to contain 300 square feet, what is the minimum amount of fence (in ft) needed to enclose the lots (include the fence across the middle)?

Answers

The minimum amount of fence needed to enclose the lots is 120ft.

Adding areas of both fields, the combined area comes about to be 600sq ft.

The dimensions of the new plot are x length and y breadth. According to the question, there is a fence in the middle with y breadth.

g(x, y, z) = xy - 600

g(x, y, z) = 2x + 3y

∇g = ∇ (xy - 600) = (y, x)

∇f = ∇(2x + 3y) = (2, 3)

Now from the Lagrange method,

∇f = λ∇g

2 = λy

3 = λx

Now to get the value of lambda,

(2/y) = (3/x) = λ

x = 1.5y

Solving now we get,

1.5y² = 600

y = 20

x = 1.5 × 20 = 30

Thus after getting (x,y) we can put the value in the function to get the answer as 120ft.

To learn more about Lagrange multipliers,

https://brainly.com/question/4609414

#SPJ4

Kaela wants to purchase a pair of shoes at Macy’s that cost $80. If Macy’s charges a
6% sales tax, how much will Kaela pay for the shoes, including tax?

Answers

Answer: $84.80

Step-by-step explanation:

The actual sales tax was 4.80 because 6% of 80 is 4.80

Then add the 4.80 to the 80 to get your total

Hope this helped :)

Help pls I'll give you brainliest and 15 points asap.
How does the graph of g(x)=4⌊x⌋ differ from the graph of f(x)=⌊x⌋?

1. Multiplying by 4 shifts the graph of g(x)=4⌊x⌋ up 4 units.
2.Multiplying by 4 shifts the graph of ​g(x)=4⌊x⌋​​ down 4 units.
3.Multiplying by 4 stretches the graph of ​g(x)=4⌊x⌋​ vertically by a factor of 4.
4.Multiplying by 4 shifts the graph of ​g(x)=4⌊x⌋​ right 4 units.

Question 2
How does the graph of g(x)=⌊x⌋−3 differ from the graph of f(x)=⌊x⌋?

1.The graph of g(x)=⌊x⌋−3 is the graph of f(x)=⌊x⌋ shifted right 3 units.
2.The graph of g(x)=⌊x⌋−3 is the graph of f(x)=⌊x⌋ shifted left 3 units.
3.The graph of g(x)=⌊x⌋−3 is the graph of f(x)=⌊x⌋ shifted up 3 units.
4.The graph of g(x)=⌊x⌋−3 is the graph of f(x)=⌊x⌋ shifted down 3 units.

Answers

Answer:

Question 1: #3 is correct.

Question 2: #4 is correct.

What is the solution set for ly-9| ≤122
O y23 ory≤21
O
-35y≤21
O y2-3 ory$21
0 35ys21

Answers

The solution set for the inequality is.

-113 ≤ y ≤ 131

What is the solution set for the inequality?

Here we have the following inequality:

|y - 9| ≤ 122

Any absolute value inequality can just be decomposed as follows:

|y - 9| ≤ 122

to:

-122 ≤  y - 9 ≤  122

Now we can solve this for y by adding 9 in all places:

-122 + 9 ≤ y - 9 + 9 ≤  122 + 9

-113 ≤ y ≤ 131

That is the solution set.

Learn more about inequalities:

https://brainly.com/question/24372553

#SPJ1

a fair coin is tossed 28 times. what is the probability that at most 25 tails occur? a) 0.99998628 b) 0.00000152 c) 0.99999848 d) 0.00001220 e) 0.01001372 f) none of the above.

Answers

The probability that at most 25 tails occur is 0.99999848.

We know that,

P(x = x) = ⁿCₓ × pˣ × [tex]q^{n-x}[/tex]

p = probability of success = 0.5

n = number of trials = 28

q = 1 - p = 0.5

The probability of the tail occurring at most 25 times can be written as

P(X ≤ 25) = P (X = 0) + P(X = 1) + ..... + P(X = 25)

Using a binomial probability calculator,

P(X ≤ 25) = 0.99999848

Therefore the probability that at most 25 tails occur is 0.99999848

To learn more about binomial probability,

https://brainly.com/question/13384135

#SPJ4

Solve the double-ended inequality:
2x-5<4x+2<2x-3

Answer: _ _ x _ _

Answer with a number in the first box, inequality in the second box, than after the x an inequality in the 3rd box and a number in the last box.

EG 5 > x > 18

Answers

The solution of the double-ended inequality is -7/2 < x < -5/2

Solving Linear Inequalities

From the question, we are to solve the doubled ended inequality.

The given inequality is

2x - 5 < 4x + 2 < 2x -3

From the inequality, we can write that

2x - 5 < 4x + 2 and 4x + 2 < 2x - 3

Now, we will solve each of the inequalities separately

Solving 2x - 5 < 4x + 2

Add 5 to both sides

2x - 5 + 5 < 4x + 2 + 5

2x < 4x + 7

Subtract 4x from both sides

2x - 4x < 4x - 4x + 7

-2x < 7

Divide both sides by -2 and reverse the sign

x > -7/2

-7/2 < x

Solving 4x + 2 < 2x - 3

Subtract 2x from both sides of the equation

4x - 2x + 2 < 2x - 2x -3

2x + 2 < -3

Subtract 2 from both sides

2x + 2 -2 < -3 - 2

2x < -5

Divide both sides by 2

x < -5/2

Putting the two solutions together, we get

-7/2 < x and x < -5/2

-7/2 < x < -5/2

Hence, the solution is -7/2 < x < -5/2

Learn more on Solving linear equations here: https://brainly.com/question/246993

#SPJ1

Bill wants to save for his son’s college expenses. He knows what tuition will cost in 18 years and knows the interest rate on the bank account. What he doesn’t know is how much to deposit each month to save enough for his son’s college expenses. Which Excel formula would you use?

Answers

The Excel formula that Bill would use is PMT.

How to illustrate the information?

From Bill's situation, he has the following information:

time = NPER = number of periods = 18 years

RATE of the bank

FV = future value = cost of tuition

PMT, is a financial function, computes the loan payment based on constant payments and a constant interest rate. The Excel PMT function is a financial function that computes loan payments based on a fixed interest rate, the number of periods, and the loan amount. The function's name is derived from the acronym "PMT," which stands for "payment."

In conclusion, the correct option is PMT.

Learn more about Excel on:

https://brainly.com/question/25879801

#SPJ1

Complete question

Bill wants to save for his son’s college expenses. He knows what tuition will cost in 18 years and knows the interest rate on the bank account. What he doesn’t know is how much to deposit each month to save enough for his son’s college expenses. Which Excel formula would you use? Choose from FV, PV, PMT, NPER, or RATE.

find the surface area of a rectangle prism that has the following deminsions.
length : 2ft
width : 14in
height : 11in
surface area : in 2

Answers

Answer:

SA(surface area) of the rectangular prism is 1,510 in inches

Step-by-step explanation:

SA=2(lh+lw+hw)

2inch=24ft

lh=264

lw=336

wh=154

755*2=1,510

What is the quotient (x2 − 9x + 20) ÷ (x − 4)? (1 point) x − 5 x − 4 x + 4 x + 5

Answers

Answer:

The guy under me is correct

Step-by-step explanation:

Answer:

x - 5

Step-by-step explanation:

Factor the expressions that are not already factored

(x^2 - 9x + 20) / (x - 4)

Cancel out x - 4 in both numerator and denominator

[tex]\frac{(x-5)(x-4)}{x-4}[/tex]

x - 5

A line segment has endpoints at (–4, –6) and (–6, 4). Which reflection will produce an image with endpoints at (4, –6) and (6, 4)?

Answers

The required expression of the reflection of the point is (-x, y).

Given that,
A line segment has endpoints at (–4, –6) and (–6, 4). Which reflection will produce an image with endpoints at (4, –6) and (6, 4) is to be determined.

What is the expression?

The expression is the relationship between variables and represented as y = ax + b is an example of a polynomial equation.

Here,
A line segment has endpoints at (–4, –6) and (–6, 4) and an image with endpoints at (4, –6) and (6, 4) is produced the equation of the reflection is given as,
(x, y) ----> (-x, y)

Thus, the required expression of the reflection of the point is (-x, y).

Learn more about expression here:

brainly.com/question/10413253

#SPJ1

a student in public administration wants to estimate the mean monthly earnings of city council members in large cities. she can tolerate a margin of error of $100 in estimating the mean. she would also prefer to report the interval estimate with a 95% level of confidence. the student found a report by the department of labor that reported a standard deviation of $1,000. what is the required sample size? (hint: round your answer up to the nearest whole number)

Answers

the required sample size for the interval estimate with a 95% level of confidence  is 385

since we are given the margin of error which is f $10 and a standard deviation which is $1,000,  and we are also given the level of confidence which is 95%

so the formula we are  referring to calculate the sample size is :

n=z(0..95)*s²/ E

here, E is the margin of error, S is the standard deviation

so z score of 0.95 is 1.96

so n = 1.96*1000^2/100

        = 384.16 ,which is almost 385

To know more about standard deviation refer to the link https://brainly.com/question/12402189?referrer=searchResults.

#SPJ4

A sample survey of 54 discount brokers showed that the mean price charged for a trade of 100shares at 50 per share was $33. 77 (AAII Journal, February 2006). The survey is conducted annually. With the historical data available, assume a known population standard deviation of $15.

a. Using the sample data, what is the margin of error associated with a 95% confidence interval?

b. Develop a 95% confidence interval for the mean price charged by discount brokers for a trade of 100 shares at $50 per share

Answers

a) The margin of error associated with a 95% confidence interval is 4.

b) The 95% confidence interval for the mean price charged by discount brokers for a trade of 100 shares at $50 per share is ($29.77,$37.77)

a) We are given with the information that the mean for 54 discount brokers is $50 for 100 shares is $33.77 Also the population standard deviation is $15.

The formula for margin of error is Z  = zα/2 σ √ n

Now we know from our z-table that value of zα/2 for 95% confidence interval is 1.96

So, the value of error margin is  zα/2 σ √ n =1.96 (15/√54) = 1.96 x 2.04

= 4

b) We know from part a that margin of error is 4.

Also, the mean price charged for a trade of 100shares at 50 per share was $33. 77

The value of confidence interval for population mean is,

x ± zα/2 σ √ n = 33.77 ± 4 = (29.77,37.77)

Thus, the 95% confidence interval for the mean price charged by discount brokers for a trade of 100 shares at $50 per share is ($29.77,$37.77)

Get to know more about Confidence Intervals and Error of Margins from here:

https://brainly.com/question/24131141

#SPJ4

3.472 divided by ____=0.112 free 25 points!!

Answers

When the divisor is missing, you divide the dividend (3.472) by the quotient (0.112)

So, we will do 3.472 divided by 0.112

3.472/0.112 = 31

Another method is multiplication. Multiplication is the opposite of division. We'd multiply 0.112 by 31.

0.112 x 31 = 3.472

In conclusion,

3.472/31 = 0.122

If you need extra help, use this video as a reference

https://youtu.be/9ui8Yh0bwCI?t=83

Answer: 3.472/31 = 0.122

Step-by-step explanation: I hope this helps:)

Choose the linear equation that is parallel to the following line y =−1/4 x − 1

Answers

Answer:

Any equation that has the same slope as this line will be parallel to it. The slope is -1/4, so whichever equation has -1/4 x in it will be parallel to the line.

At the bakery, they have 4 and a 1/8 pies for sale. They sell 1⁄4 of a pie for$5. How many pieces do they have to sell, and how much money will they make?

Answers

The number of pieces the bakery will sell is 16 and a half. The total amount of money they will make is $82.5.

What is the number of pieces of pies to be sold?

To get the number of pieces of pies that they have to sell, we need to first determine how many 1/4 pies are in 4 and 8 pieces of bread. Going by that, we can see that:

1/4x of 4 = 16.

Also, 1/4x of 1/8 = 1/2.

So, the total number of pies that they have to sell is 16 and a half.

Now if 1/4 of a pie costs $5, then 4 1/8 will be equal to 33/8 × 5 ÷ 1/4.

This is equal to 165/8 × 4 = $82.5

So, the total amount of money that the bakery will make will be $82.5.

Learn more about fractions here:

https://brainly.com/question/78672

#SPJ1

The demand function for a certain make of replacement cartridges for a water purifier is given by the equation p=−0. 01x2−0. 1x+21 where p is the unit price in dollars and xis the quantity demanded each week, measured in units of a thousand. Determine the consumers' surplus if the market price is set at $9/cartridge. (Round your answer to two decimal places. )

Answers

The demand function for a certain is given by the equation p=−0. 01x2−0. 1x+21 then the determined values will be −50.905557, −0.427737,41.333295.

From the given values,

R=px

9 = (−0. 01x2−0. 1x+21) x

0.01x3+0.1x2−21x−9=0

By reducing the equation,

X = −50.905557, −0.427737,41.333295.

Therefore, the determined values will be −50.905557, −0.427737,41.333295.

Learn more about Equation here

https://brainly.com/question/15707224

#SPJ4

everyone can someone help me here please (complete solution and graph)​

Answers

Answer:

1: x^2+y^2-6x-4y-3=0

2: x^2+y^2-10x+4y=0

3: x^2+y^2-10x+2y+1=0

a tire manufacturer wishes to investigate the tread life of its tires. a sample of 10 tires driven 50,000 miles revealed a sample mean of 0.32 inches of tread remaining with a sample standard deviation of 0.09 inches. construct a 95% confidence interval for the population mean.

Answers

The confidence interval for the population mean is 0.2896 , 0.3505)

The sample mean is x = 0.32

The sample standard deviation is s = 0.09

The sample size is n = 10

The population standard deviation is unknown and sample size is atleast 10.

Thus use the t distribution.

Replacing the values in the following formula, determine the 95% confidence interval.

95% C.I = x ± t₀.₉₅,ₙ₋₁ . Sm

= x ± t₀.₀₅/₂,₁₀₋₁ . s/√n

= 0.32 ± 2.03 . 0.09/√10

= (0.2896 , 0.3505)

Hence the confidence interval for the population mean is 0.2896 , 0.3505)

Learn more about Confidence Interval here:

brainly.com/question/17097944

#SPJ4

A soft drink manufacturer has a daily production cost of C = 70,000 - 120x + 0.055x ^ 2 , where C is the total cost (in dollars) and x is the number of units produced. How many units should they produce each day to yield a minimum cost?
Show all work please.

Answers

The minimum cost will be 1091 units.

Define Vertex Formula.

When the graph crosses its symmetry axes, the vertex formula aids in determining the vertex coordinate of a parabola. The vertex point is often represented by (h, k).

We are aware that a parabola's conventional equation is y=ax²+bx+c. The vertex in this case should be near the base of the U-shaped curve if the coefficient of x² is positive. The vertex should be at the peak of the U-shaped curve if the coefficient of x² is negative.

Given expression is,

C = 70,000 - 120x + 0.055x²

For this equation, the graph will be parabola opening upward.

For that, the vertex will be the minimum cost

x = (-b) / 2a

Here, a = 0.055 , b = -120

Now, plug in the values and find 'x'

minimum cost (x) = ( -(-120) ) / ( 2 * 0.055 )

                            = 120 / 0.11

                            = 120 / 0.11

                            = 1090.90  or 1091 units

Therefore, the minimum cost will be 1091 units.

To read more about Vertex Formula.

https://brainly.com/question/525947

#SPJ13

Please help with algebra. Find the formula for an exponential equation that passes through the points, (0,2) and (1,6). The exponential equation should be of the form y=abx.

Answers

The exponential function has an equation of f(x) = 2(3)^x

How to determine the equation of the exponential function?

From the question, we have the following points that can be used in our computation:

(0, 2) and (1,6)

Also, we understand that

Function type = Exponential function

An exponential function is represented as

f(x) = ab^x

At point (0, 2), we have

2 = ab^0

Evaluate

a = 2

Substitute a = 2 in f(x) = ab^x

f(x) = 2b^x

At point (1, 6), we have

6 = 2b^1

This gives

2b = 6

Divide

b = 3

So, we have

f(x) = 2(3)^x

Hence, the equation of the exponential function is f(x) = 2(3)^x

Read more about exponential function at

https://brainly.com/question/2456547

#SPJ1

Answer it now bois now

Answers

Answer:

Answer is [tex](\frac{4}{-1})[/tex] option D is correct

13) Mr. Harrington wants to buy a new TV for $500. He has a 10% off coupon and knows the tax
be 13.5%. How much money will he need to save to buy the TV?
a) $510.75
c) $450
b) $432.50
d) $567.50

Answers

Mr. Harrington will need $510.75  to buy the TV

The correct answer is an option (a)

In this question, Mr. Harrington wants to buy a new TV for $500

He has a 10% off coupon.

After applying this coupon, the cost of TV would be,

500 - (10 percent of $500)

= 500 -[ (10/100) * 500]

= 500 - 50

= 450

He knows the tax be 13.5%.

13.5 percent of 450 would be,

(13.5/100) * 450

= 60.75

So, the amount he will need to save to buy the TV,

$450 + $60.75 = $510.75

Therefore, Mr. Harrington will need $510.75  to buy the TV

The correct answer is an option (a)

Learn more about the percentage here:

https://brainly.com/question/16797504

#SPJ9

(Help!)

#8 The tables represent some points on the graphs of lines m and n. Which system of equations is represented by lines m and n?
(A) −6x − y = 8 −3x + 2y = 9
(B)6x + y = −8 3x − 2y = 9
(C) 6x − y = −8 −3x − 2y = 9
(D) 6x − y = 8 3x + 2y = 9

#9 A produce company sells crates filled with a mixture of apples and oranges during the holiday season to grocery stores. Each crate contains a total of 72 pieces of fruit. Suppose that a crate has 8 more apples than oranges.

Which system of linear equations can be used to determine the number of pieces of each kind of fruit in the crate?
(A) a + o = 72 o = a + 8
(B) 8a + o = 72 a + o = 8
(C) a + o = 72 a = o + 8
(D) a + 8o = 72 a + o = 8

#10 The tables represent some points on the graphs of lines r and s. Which system of equations is represented by lines r and s?

(A) −4x + 3y = 12 2x − 5y = −10
(B) −4x − 3y = −12 −2x + 5y = 10
(C) 4x + 3y = −12 2x − 5y = 10
(D) 4x − 3y = −12 −2x − 5y = 10

Answers

The linear functions representing the systems of equations in each case are given as follows:

8. (D) 6x − y = 8 3x + 2y = 9

9. (C) a + o = 72 a = o + 8.

10. (C) 4x + 3y = −12 2x − 5y = 10.

What is a linear function?

The slope-intercept representation of a linear function is presented by the rule as follows:

y = mx + b

The coefficients of the function and their meaning are listed as follows:

m is the slope of the function, representing the change in the output variable y when the input variable x increases by one.b is the y-intercept of the function, being the numeric value of the function when the input variable x assumes a value of 0.

For the line m, we have that when x increases by 6, y increases by 36, hence the slope is:

m = 36/6 = 6.


Considering the positive slope of 6, at x = 0, the numeric value is 12 less than the numeric value at x = 2, hence the intercept is:

b = 4 - 2 x 6 = -8.

Hence the equation is:

y = 6x - 8.

6x - y - 8.

Thus option d is correct.

For item 9, we use the variables a and o. The total is of 72, hence:

a + o = 72.

Suppose that a crate has 8 more apples than oranges, hence:

a = o + 8.

Thus option c is correct.

For line r in item 10, when x increases by 3, y decays by 4, hence the slope is:

m = -4/3.

When x = 0, y = -4, hence the intercept is:

b = -4.

Thus the equation is:

y = -4x/3 - 4

3y = -4x - 4.

4x + 3y = -12.

Meaning that option c is correct.

More can be learned about linear functions at https://brainly.com/question/24808124

#SPJ1

The perimeter of a rectangle can be found using the equation P = 2L + 2W, where P is the perimeter, L is the length, and W is the width of the rectangle.
Solve the equation for the variable w
When you have done that
state the value of 'w' if the perimeter of the rectangle is equal to 37 feet and the length is equal to 4 feet.
hurrryyyy pls

Answers

Answer:

14.5 feet

Step-by-step explanation:

To make  subject as 'w' and solving:

     To make the subject as 'w', isolate w.

To isolate 'w', subtract 2L from both sides .

              2L + 2W =P

                      2W = P - 2L

Now, divide both sides by 2.

                        [tex]\sf W = \dfrac{P-2L}{2}\\\\W =\dfrac{P}{2}-\dfrac{2L}{2}\\\\ \boxed{W = \dfrac{P}{2}-L}[/tex]

   P = 37 feet

   L = 4 feet

           [tex]\sf W =\dfrac{37}{2}-4\\\\[/tex]

                = 18.5 - 4

           W = 14.5 feet

Round 45.4673
off each number
to two (2) Decimal Place

Answers

Answer:

45.47

Step-by-step explanation:

Could someone help quick

Answers

The constant of proportionality is known to be the ratio of two proportional values that is said to be in a constant value.

What is the constant of proportionality?

The constant of proportionality is a term that is defined as the rate or the ratio of two proportional values which are known to be at the same value.

constant of proportionality = 2.5

Given  :  y = 2.5x.

To find : What is constant of Proportionality in the equation.

Solution : We have given

y = 2.5x.

On comparing with equation of proportionality : y = kx,

Where,   k is the constant of proportionality.

On comparing y = 2.5 x  by y = kx .

We get  k = 2.5

So, constant of proportionality = 2.5

Therefore, constant of proportionality = 2.5

To learn more about constant of proportionality refer

https://brainly.com/question/28413384

#SPJ1

The value of constant proportionality is 2.5. The constant of proportionality is known to be the ratio of two proportional values that is said to be in a constant value.

What is the constant of proportionality?

The constant of proportionality is a term that is defined as the rate or the ratio of two proportional values which are known to be at the same value.

constant of proportionality = 2.5

Given  :  y = 2.5x.

To find : What is constant of Proportionality in the equation.

Solution : We have given

y = 2.5x.

On comparing with equation of proportionality : y = kx,

Where,   k is the constant of proportionality.

On comparing y = 2.5 x  by y = kx .

We get  k = 2.5

So, constant of proportionality = 2.5

Therefore, constant of proportionality = 2.5

To learn more about constant of proportionality refer

brainly.com/question/28413384

#SPJ1

In the image below, the measure of angle 3 is equal to the sum of the measures of angle 6 and what other angle?

Answers

For the given figure, angle 3 will be equal to angle 2.

According to the question,

We have the following information:

We have a figure where some angles are marked in form of numbers and  the measure of angle 3 is equal to the sum of the measures of angle 6.

Now, we know that vertically opposite angles are equal and angle 1 and angle 4 are vertically opposite angles like angle 2 and angle 3.

(More to know: the sum of three angles of a triangle is equal to 180°.)

Hence, for the given figure, angle 3 will be equal to angle 2.

To know more about angle here

https://brainly.com/question/68367

#SPJ1

help meeeeeeeee pleaseeeee rn rnnnn!!!!!!!!!!!!!!!!!!!!!!!!!!!!!!!
help meeeeeeeee pleaseeeee rn rnnnn!!!!!!!!!!!!!!!!!!!!!!!!!!!!!!!
help meeeeeeeee pleaseeeee rn rnnnn!!!!!!!!!!!!!!!!!!!!!!!!!!!!!!!
help meeeeeeeee pleaseeeee rn rnnnn!!!!!!!!!!!!!!!!!!!!!!!!!!!!!!!

Answers

The number of bicycles that must be manufactured to minimize the cost of 250 and the minimum cost is $11,500.

In the given question,

The cost C in dollars of manufacturing x bicyles at a production plant is given by the function shown below;

C(x) = 3x^2-1500x+199,000

(a) In the given question we have to find the number of bicyles that must be  manufactured to minimize the cost.

The given function is

C(x) = 3x^2-1500x+199,000

To find the maximum or minimum value we firstly find the value of f'(x).

C'(x) = 6x-1500

Now put f'(x)=0

6x-1500=0

Add 1500 on both side, we get

6x=1500

Divide by 6 on both side we get

x=250

Hence, the number of bicycles that must be manufactured to minimize the cost of 250.

(b) Now finding the value of function at x=250

C(250) = 3(250)^2-1500*250+199,000

C(250) = 3*62,500-375,000+199,000

C(250) = 187,500-375,000+199,000

C(250) = 11,500

The minimum cost is $11,500.

To learn more about maximim and minimum value of quadratic equation link is here

brainly.com/question/28769828

#SPJ1

is the number 1.41421356 rational

Answers

The number 1.41421356 is not a rational number.

What is a rational number?

A number that can be expressed as a ratio is called a rational number. This indicates that it may be expressed as a fraction, where the numerator (the number on top) and denominator (the number on the bottom) are both whole numbers.

The given number is 1.41421356

The square root of 2 has the approximate value of 1.41421356

The number is non-terminating and non-repeating decimals.

That's why the given number is not a rational number.

To learn more about rational numbers

https://brainly.com/question/7563619

#SPJ1

examples of linear equation

Answers

Any equation where the highest degree (highest exponent) is 1. Because of this, the slope goes in a uniform rate. Examples:

Y=x^1 +b
Y=4x+3
Y=-5x+2/3
Y=4.5x+2
Y=x
Other Questions
when a former partner of a registered public accounting firm who left the firm two years ago accepts a financial reporting oversight role at an issuer audit client, the independence of the registered public accounting firm is considered impaired unless: HELP!! Find three consecutive odd integers that twice the sum of the second and the third is 43 more than three times the first help please for brianlest??? a television commercial for senseo brand of coffee maker that can make coffee similar to what consumers purchase at coffee shops (e.g., starbucks) encourages viewers to remember the experience of drinking their favorite coffee drinks at these types of shops. this coffee maker allows consumers to experience that sensation at home. what is this marketer trying to encourage? 2 balls ball 1 and ball 2 are each thrown horizontally from the same height above the ground. ball 2 has a greater initial velocity after leaving the thrower's hand than ball 1. if air resistance is negligible how do the accelerations of each ball and the time it takes each of them to hit the ground compare? Which of the following graphs represent an object at rest? (There could be more than one correct cho 0' 0' 0 0 graph a graph b graph c graph d describe the most likely effect of gasdermin pore formation on water balance in the cell in a hypotonic environment. why do different types of matter have different requirements to phase change? If you need to know what the colors on a circle graph mean, you can look attheA. legendB. pointsC. barsD. title interest rates on 4-year treasury securities are currently 5.75%, while 6-year treasury securities yield 7.25%. the data has been collected in the microsoft excel online file below. open the spreadsheet and perform the required analysis to answer the question below.if the pure expectations theory is correct, what does the market believe that 2-year securities will be yielding 4 years from now? calculate the yield using a geometric average. do not round your intermediate calculations. round your answer to two decimal places. To settle all local and religious disputes in Judea. The Romans allowed the Jewish court, called the A. Order B. Senate C. Parliament D. Sanhedrin The american stock market collapsed on october 29,1929 , marking the beginning of the great depression? what is that day known as? some help me pls with thisSolve the equation: r - 535 = 175 blink appliances plans to order microwaves and stoves. each microwave requires 2 hours to unpack and set up in the storeroom, and each stove requires 4 hours. the storeroom space is limited to 68 items. the budget of the store allows only 244 hours of employee time for unpacking and setup. microwaves yields a profit of $128 each, and stoves yields a profit of $174 each. how many of each should the store order to maximize profit? organizational learning works best when there is integrated thinking and acting at all levels of the organization, according to peter senge. what is 100200 need help xyz title company is closing a transaction for bob buyer and discovers a mortgage still of record from community bank. in fact, bob paid the mortgage off six months earlier. bob buyer gets a call from his lender, boa, that they can't close the loan because of the community bank mortgage. bob loses a month's time and the interest rate on his loan goes up a half point while community bank records the satisfaction. what are the rights of the parties? In which situation would the use of a vector quantity be most appropriate? (1 point)determining the net force that is acting on an objectdetermining the high and low temperatures on any given daydetermining whether it takes more time to melt ice or to melt butterdetermining whether a van is moving faster than the speed limit Derek's team is struggling to come to a consensus because several people are unwilling to share their thoughts. What would be the BEST question for the group to ask themselves to avoid groupthink?A: How is this situation similar to others we've encountered?B: What about the group could be making people feel unsafe?C: How would this situation look different if we had more money?D: Can we get rid of the people who aren't willing to speak up? what is the improvement in the frequency response of the modified equalizer and what benefits does that bring to the system?